You should know this number theory result -- Bertrand's Postulate

  Рет қаралды 44,281

Michael Penn

Michael Penn

Жыл бұрын

🌟Support the channel🌟
Patreon: / michaelpennmath
Merch: teespring.com/stores/michael-...
My amazon shop: www.amazon.com/shop/michaelpenn
🟢 Discord: / discord
🌟my other channels🌟
Course videos: / @mathmajor
non-math podcast: / @thepennpavpodcast7878
🌟My Links🌟
Personal Website: www.michael-penn.net
Instagram: / melp2718
Randolph College Math: www.randolphcollege.edu/mathem...
Research Gate profile: www.researchgate.net/profile/...
Google Scholar profile: scholar.google.com/citations?...
🌟How I make Thumbnails🌟
Canva: partner.canva.com/c/3036853/6...
Color Pallet: coolors.co/?ref=61d217df7d705...
🌟Suggest a problem🌟
forms.gle/ea7Pw7HcKePGB4my5

Пікірлер: 116
@jackhandma1011
@jackhandma1011 Жыл бұрын
I still can't get over the fact that a 17 year old proved the same thing about pseudoprimes. It's like a Gauss-type feat, but happened in our era.
@sk8erJG95
@sk8erJG95 Жыл бұрын
It is impressive, but he's not just any 17-year old. His parents are both number theorists at Bloomington who are Princeton/Harvard-educated, Michael Larsen and Ayelet Lindenstrauss. His uncle is Elon Lindenstrauss, a Fields Medalist who works on applications of Ergodic theory to number theory. His grandpa was also a mathematician that made lots of progress in functional analysis and banach spaces, Joram Lindenstrauss. He has a world of mathematical support and encouragement that most of us couldn't even dream of. I'm glad he's using it well!
@Gaeisok
@Gaeisok Жыл бұрын
@@sk8erJG95 interesting! If I had all that support I still wouldn't be able to solve it but ut definitely must have helped him
@lgooch
@lgooch Жыл бұрын
@@sk8erJG95 wow, that’s very interesting
@r.maelstrom4810
@r.maelstrom4810 Жыл бұрын
I have looked into his paper. I can't believe it. Not only by the pure and raw computations done, but also due to the very advanced concepts (difficult to grasp for even postgraduate mathematicians) he uses joyfully. High abstract algebra, analytic number theory of very advanced level, topological and group theory... I can't believe it. It's far beyond any math olympic medallist level of math.
@tyrjilvincef9507
@tyrjilvincef9507 Жыл бұрын
@@sk8erJG95 "More mathematical support" And genes tbh.
@justinbliss3507
@justinbliss3507 Жыл бұрын
Hey guys, Justin the editor here, it seems I left in an error meant to be cut at about 14:29 (the correct part starts at 16:47). I'm fixing it with the youtube editor, but it may take some time for the changes to go live. Sorry for the inconvenience!
@actions-speak
@actions-speak Жыл бұрын
Thanks for all your hard work in bringing us so much great math content!
@richardsandmeyer4431
@richardsandmeyer4431 Жыл бұрын
The original implied that square root of 2 is less than 2/3, so it was obvious that something was wrong. Thanks for correcting it.
@matematicacommarcospaulo
@matematicacommarcospaulo Жыл бұрын
Justin.... Could erase part of a video without re-upload it? If yes, teach me how
@deadfish3789
@deadfish3789 Жыл бұрын
@@richardsandmeyer4431 Does it? How? I suppose the splitting makes no sense when sqrt(2n)>2n/3, which occurs when n2048, so it isn't actually relevant. I hope the entire segment isn't being removed - while obviously wrong (the product of primes needn't be greater since some may appear more than once in (2n n), it needs replacing with something, since the board is much fuller at 16:47 than 14:29.
@ybn9861
@ybn9861 Жыл бұрын
is there a fixed version now?
@stanleydodds9
@stanleydodds9 Жыл бұрын
There's a big error in this proof. You've missed a whole step by using an incorrect inequality at 14:20. 2n choose n is not necessarily at most the product of primes up to 2n, because primes can have exponents bigger than 1 in the factorisation. For example, 10 choose 5 is divisible by 2^2 and 3^2. A correct proof would use Legendre's formula to say that the exponent of a prime p dividing 2n choose n is at most log_p(2n). This way, you can bound each prime *power* in the factorisation by p^(log_p(2n)) = 2n. But in the incorrect proof you gave, you simple said each prime has exponent at most 1 (which is untrue) and then simply used the very weak bound that p
@Bazzzzz93
@Bazzzzz93 Жыл бұрын
Yes
@Neodynium.the_permanent_magnet
@Neodynium.the_permanent_magnet Жыл бұрын
See the comment from Justin Bliss below...
@Mmmm1ch43l
@Mmmm1ch43l Жыл бұрын
@@Neodynium.the_permanent_magnet but the comment doesn't address this issue at all?
@monishrules6580
@monishrules6580 Ай бұрын
I was confused on that
@gregghutchence
@gregghutchence Жыл бұрын
Chebyshev said it, And I say it again, There is always a prime Between n and 2n. - Erdos
@theimperator1327
@theimperator1327 Ай бұрын
A misattribution, he never said it.
@ahoj7720
@ahoj7720 Жыл бұрын
At 18:15, you can do slightly better and show that 2n choose n is greater or equal to 4^n/(2n) [consider the 2n-th row of Pascal's triangle, which sums to 4^n, add separately the first and last ones and compare the central binomial to all other terms]. In the last inequality at 18:54, which will lead to contradiction, the factor 2n+1 is then replaced by 2n. To conclude, let 2n=u^2. The inequality becomes 2^((u^2)/3) 800 (instead of your 2048). Same trick as you use for the first 800 integers. [In fact u>=31 and n>480]
@MoreDenseThanRationals
@MoreDenseThanRationals Жыл бұрын
Erdos came up with this proof when he was 19!
@boristerbeek319
@boristerbeek319 Жыл бұрын
That's quite old actually 😛
@bjornfeuerbacher5514
@bjornfeuerbacher5514 Жыл бұрын
Erdos got more than 121 000 000 000 000 000 years old? :O ;)
@jimschneider799
@jimschneider799 Жыл бұрын
@1:00 - Since 4^n is a power of 2, and any product that includes an odd number is not a power of 2, that product will be less than 4^n if it's less than or equal to 4^n
@galoomba5559
@galoomba5559 Жыл бұрын
I don't get 14:03. Shouldn't a number be _greater_ or equal to the product of primes that divide it, as some of the primes can appear in the factorisation multiple times?
@matteobianchi2038
@matteobianchi2038 Жыл бұрын
I was thinking the same thing
@saroshadenwalla398
@saroshadenwalla398 Жыл бұрын
So what he should have written at that point is that 2nCn is equal to the product of p^R over primes less than or equal to 2n/3 where R is the highest power of p that divides 2n. Then if p>(2n)^0.5 there is at most one power of p that divides 2nCn, and for any prime p, p^R is less than or equal to 2n. So the product of p^R over p less than or equal to 2n/3 is then split into two products of p^R for p less than or equal to (2n)^0.5 and for p>(2n)^0.5. The first product can be upper bounded by (2n)^((2n)^0.5) and the second product can be upper bounded by the product of p for p less than or equal to 2n/3.
@stanleydodds9
@stanleydodds9 Жыл бұрын
Yeah, he actually made quite a big mistake here, and there's a whole other part of the proof that needs to be done to show the inequality. What you actually want to do here is use Legendre's formula to show that the exponent of p in the factorisation of 2n choose n is at most log_p(2n), but this is definitely non-trivial and what he wrote is incorrect (some primes definitely do appear multiple times). If you do this correctly though, then you can say that 2n choose n is at most the product of p to the power of this maximum exponent, log_p(2n), for all primes p in the range. Then you can see that p^(log_p(2n)) = 2n, so each of these prime power terms is at most 2n, which is what he writes next. Then you can continue following the proof.
@jaddaj5881
@jaddaj5881 Жыл бұрын
I was looking in the comments because I had the same question
@pratikdash10
@pratikdash10 Жыл бұрын
Ah !! I'm not alone :D. Also, don't understand how sqrt(2)*n is less than 2n/3.
@jimallysonnevado3973
@jimallysonnevado3973 Жыл бұрын
14:20 how is the central binomial coefficient less than the product of all primes that divides it? Isn't it that 2n choose n contains all primes that divide it as a factor ad the opposite must be true?
@aweebthatlovesmath4220
@aweebthatlovesmath4220 Жыл бұрын
I was looking for this proof thank you!
@johndwolynetz6495
@johndwolynetz6495 Жыл бұрын
don't wory he forgot to add the exponents in the left product 1
@romajimamulo
@romajimamulo Жыл бұрын
12:13 Justin did not have that formula on the screen. Is it in the description?
@ShenghuiYang
@ShenghuiYang Жыл бұрын
It seems the binomial (2n,n) thing follows Kummer theorem. Great video!
@digxx
@digxx Жыл бұрын
For everyone who is wondering about the missing steps at 14:30, here they are. Since we know that there is no prime p>2n/3 in the product (2n,n), we have (2n,n)=\prod_{p
@deadfish3789
@deadfish3789 Жыл бұрын
I think you've overcomplicated it for the first product. All you need to point out is that obviously p^(k_p)
@digxx
@digxx Жыл бұрын
@@deadfish3789 Why is p^(k_p) a factor of 2n? Don't we just know the info about (2n,n)? I mean, p^{k_p} is, by definition, a factor of (2n,n), so how should it divide 2n in general. Or do you mean (2n)! ?
@anon6514
@anon6514 Жыл бұрын
8:13 Damn it Michael, making me do the leg work to follow along :) Prove that Choose(2k+1,k) =1 I see why 4 is used as the bound now... lim { (4k+6)/(k+2) } = 4
@DarinBrownSJDCMath
@DarinBrownSJDCMath Жыл бұрын
Look at Pascal's triangle -- the sum of the entries in row (2k + 1) is 2^(2k + 1) = 2 * 4^k, so half the sum is 4^k.
@xyzx1234
@xyzx1234 Жыл бұрын
@15:13: Maybe I'm missing something here, but what's the point of breaking the product into two parts and then upper-bounding one of the parts by the original product? Notice the term in red is exactly the left-hand side of the inequality. Doesn't the lemma give prod_{p
@deadfish3789
@deadfish3789 Жыл бұрын
The original inequality (2n n)
@ssvemuri
@ssvemuri Жыл бұрын
Totally love your white-boarding and tutorial style of teaching. It makes me yearn to go back to schoo. Gripes: this result if obviously very appealing. But thinking of the lemmas is hard. Why one earth (or how) does one figure out the right lemmas to go after? I realize theorem construction is hard, but some pointers will be useful.
@goodplacetostop2973
@goodplacetostop2973 Жыл бұрын
21:40
@brahimsebbata9036
@brahimsebbata9036 Жыл бұрын
Thanks good explication
@supratimsantra5413
@supratimsantra5413 5 ай бұрын
Super impressive teaching sir....from our country India we appreciate you the most over others lectures
@Anonymous-zp4hb
@Anonymous-zp4hb Жыл бұрын
Happy Birthday, Erdős!
@ramuk1933
@ramuk1933 Жыл бұрын
I don't see how the product of the primes could ever be equal to 4^n, as for n>2, 3 would be a factor, thus it couldn't be any power of 4.
@gheffz
@gheffz Жыл бұрын
Thank you... that's was a bit of a marathon to get to the "good place to stop".
@benjamin7853
@benjamin7853 Жыл бұрын
8:20 You don't need induction to prove, just realize that 2k+1 choose k = 2k+1 choose k+1 and the sum of those two
@pixel_informatic9717
@pixel_informatic9717 Жыл бұрын
4:22 Then shouldn't you suppose that k is greater or equal to 2 so it holds everytime ?
@jimallysonnevado3973
@jimallysonnevado3973 Жыл бұрын
I’m pretty sure I had watched a video where he used this postulate perhaps in a math contest but I can’t really remember it. Can anyone here send the link??
@rema_style
@rema_style 7 ай бұрын
Could you proof that there is prime n^3
@johnchessant3012
@johnchessant3012 Жыл бұрын
14:39 Isn't 2n/3 less than sqrt(2)*n? Why are we breaking up the product like this?
@bjornfeuerbacher5514
@bjornfeuerbacher5514 Жыл бұрын
Note the correction starting at about 16:45.
@johnnicholson8811
@johnnicholson8811 Жыл бұрын
Michael, can you explain how Ramanujan did his proof and Ramanujan primes?
@oida10000
@oida10000 Жыл бұрын
Given the examples and the proof could we conclude Product of primes less or equal n
@BrollyyLSSJ
@BrollyyLSSJ Жыл бұрын
I don't see the reason really. You can easily get rid of the equals sign, since 4^n is never a primorial for n >= 2.
@MarsAnonymous
@MarsAnonymous Жыл бұрын
Maybe the lemma was originally meant to be for all n >= 0? In this case, the product of all primes less then or equal to 0 is actually exactly 4^0 = 1. For n = 1, the product is strictly less than 4^n, and remains so.
@YoutubeBS
@YoutubeBS Жыл бұрын
Actually, this product is ≤4^(n-1)
@thesecondderivative8967
@thesecondderivative8967 Жыл бұрын
8:44 Can someone explain why we needed to prove it strictly using mathematical induction? Why wasn't the first case enough?
@schweinmachtbree1013
@schweinmachtbree1013 Жыл бұрын
If you're asking why the first of the two inductive steps wasn't enough, it's because the first inductive step shows that n ⇒ n+1 for n odd while the second inductive step shows that n ⇒ n+1 for n even, so we need both to account for all values of n
@rayannimour5196
@rayannimour5196 Жыл бұрын
it's soo cool, i wish we had more math computations in my contry(Algeria) so that we could learn if kind of tricks...even teacher in my college don't know about this postulate.i think i should go to another Country to finish my studies XD
@GeneralVimes
@GeneralVimes 8 ай бұрын
Interesting, which is the lowest n>3 such that between n and 2n exists only one prime
@johns.8246
@johns.8246 Жыл бұрын
This is the 3rd video I've seen attempting to explain the proof. I'm clueless as always. At 16:52, how do you show the inequality doesn't hold for n>2048. Also, does that imply there are NO primes p from 2
@thesecondderivative8967
@thesecondderivative8967 Жыл бұрын
I think brute force... I haven't tried it yet though.
@deadfish3789
@deadfish3789 Жыл бұрын
You can't do brute force - there are infinitely many integers above 2048. However, I'm having a lot of trouble proving it. I've proven it for n=2048, and am trying to show that the sequence a_n = (2n+1)(2n)^sqrt(2n)/4^(n/3) is decreasing by showing a_n/a_{n+1} (2^5+2^3)2^6+39 > 39(2^6+1). Then 4^2048/3 = 2^(2^12/3) > 2^(13(2^6+1)) > 2^(12(2^6+1))+2^(12(2^6)) = 2^(12(2^6))(2^12+1) = (2*2048)^sqrt(2*2048)(2*2048+1) as required. The proof said that if all the prime factors are at most 2n/3, then n=2048, then there is at least one prime >2n/3. There may still be some prime factors 2
@stans1327
@stans1327 2 ай бұрын
why do we break into parts the product in 14:30? Can't we just write that by lemma this product is < than 4^(2n/3)?
@romaindemarly127
@romaindemarly127 Жыл бұрын
As others have stated, there seem to be a step missing at around 14:53 and you can sense it's missing something because when you break the product down you actually end having the left hand side lower than (2n)^{\sqrt{2n}} but you say the right hand side is lower than the original product?! you just end up with the extra (2n)^{\sqrt{2n}} factor for no reason. your inequality would end up be 4^{n/3} \leqslant 2n+1 which is false from 6 upward, a lot less work to do than to check up to 2048. The extra work probably comes from the initial inequality that doesn't hold because of prime exponents bigger than 1.
@digxx
@digxx Жыл бұрын
That's because (2n,n)
@danteruizampolini6319
@danteruizampolini6319 Жыл бұрын
yeah exactly what i was wondering lol
@charlesbarrow803
@charlesbarrow803 Жыл бұрын
So we have a hint of one of the questions in 2048 maths challenges
@user-sh1ce3yw9f
@user-sh1ce3yw9f Жыл бұрын
12:11 wheres the formula
@khoozu7802
@khoozu7802 Жыл бұрын
It is called Legendre's formula which is the exponent of prime p in n!
@imcwaszec937
@imcwaszec937 Жыл бұрын
For every natural number n there is prime number p such that n
@antonioalbeldaochoa4775
@antonioalbeldaochoa4775 Жыл бұрын
4:53 you say 2k is not prime, then include 2k-1 as if it were to be prime, but it might not be, eg: k=13
@wernergamper6200
@wernergamper6200 Жыл бұрын
I always check the comments for errors in the video before watching.
@user-sh1ce3yw9f
@user-sh1ce3yw9f Жыл бұрын
14:30 - 16:48 seems to be a false version of the next part...
@Happy_Abe
@Happy_Abe Жыл бұрын
Can you remake this video with a correct proof? As many in the comments say there are mistakes and I’m very confused here
@innokentiyromanchenko1450
@innokentiyromanchenko1450 Жыл бұрын
seems like these product less than e^n
@robert-skibelo
@robert-skibelo Жыл бұрын
Fascinating topic, even if others smarter than me have picked a few holes in your proof. I won't criticise you for not attempting the correct pronunciation of Bertrand since I can't do it myself. The French R is impossible.
@rafael7696
@rafael7696 Жыл бұрын
I think it can be explain better
@BrollyyLSSJ
@BrollyyLSSJ Жыл бұрын
There's a small error in the first induction step of the lemma - for k =1, 2k is prime, so Prod(p
@saroshadenwalla398
@saroshadenwalla398 Жыл бұрын
He already covered the case k=1 in the base case when he looked at n=2 and 3. So in the inductive step you can take k>1
@BrollyyLSSJ
@BrollyyLSSJ Жыл бұрын
You're right, the induction hypothesis for this step should've specified k > 1, not k >= 1.
@MasterHigure
@MasterHigure Жыл бұрын
Erdös' classic proof. It's a beauty.
@euanthomas3423
@euanthomas3423 Жыл бұрын
How would anyone come up with this proof? Number theory is hard.
@jacoboribilik3253
@jacoboribilik3253 Жыл бұрын
Thinking constantly about it, talent, trial&error and a bunch of practice.
@mscandizzo1
@mscandizzo1 Жыл бұрын
This proof skips too many skips for me to follow it.
@artsmith1347
@artsmith1347 Жыл бұрын
Bertrand's *_postulate_* seems like a misnomer. Wikipedia says "Bertrand's postulate is a theorem" I was taught to make a large distinction between a postulate and a theorem. Wiktionary says a postulate is "Something assumed without proof as being self-evident or generally accepted." The wiki article says it started out as a *_conjecture_* in 1845 by Joseph Bertrand. So it seems it never was a "postulate." The wiki article continues: "His conjecture was completely proved by Chebyshev ... in 1852" So after it was proved, it still wasn't a "postulate." The name Bertrand's *_postulate_* seems to continue more than a century of imprecise language. Not that this channel can fix this problem that seems to be a tradition: I was surprised at 0:55 that this video would embark on a *_proof_* of the *_postulate_* .
@MyOneFiftiethOfADollar
@MyOneFiftiethOfADollar Жыл бұрын
Words are our servants, not our masters. Except maybe in your world.
@artsmith1347
@artsmith1347 Жыл бұрын
@@MyOneFiftiethOfADollar Mathematics prides itself on precision of language, except maybe unless the name Bertrand is "more equal than others." There doesn't seem to be an "Euler's postulate." Perhaps Euler was too busy producing results to be self-aggrandizing.
@sk8erJG95
@sk8erJG95 Жыл бұрын
Postulate is the correct term because of how Bertrand used it. You can find his paper "Mémoire sur le nombre de valeurs que peut prendre une fonction quand on y permute les lettres qu'elle renferme." on Wikipedia. He was proving another theorem and assumed there was a prime between n/2 and n-2 to do so. Hence, he used it as a postulate. A specific quote from the paper: "The lower limit that we have just given, according to M. Cauchy, for our numbers of values ​​of a function, is not higher than one can find. Except for four-letter functions that can have only two values, the indices of an n-letter function can never be both greater than 2 and less than n. To prove this proposition, I will assume as a fact that, for any number n greater than 6, there always exists a prime number, at least, between n-2 and n/2. this proposition is true for all numbers less than 6 million, and there is every reason to believe that it is true generally" Hence, when Chebyshev proved this a few years later, he described it as "Bertrand's Postulate".
@artsmith1347
@artsmith1347 Жыл бұрын
@@sk8erJG95 Thank you for the good info. My conclusion is that we can blame Chebyshev -- and those who didn't dare to to correct Chebyshev's mischaracterization -- for the sloppy language. Though no one knew it at the time, it was to become a theorem. " 'I will assume as a fact' --for my present purpose" was properly stated to be an assumption.
@DarinBrownSJDCMath
@DarinBrownSJDCMath Жыл бұрын
Culture and tradition sometimes trump precision.
@user-hq7hi2sl2o
@user-hq7hi2sl2o Жыл бұрын
asnwer=2p
@GirishManjunathMusic
@GirishManjunathMusic Жыл бұрын
Isn't √2·n > n > ⅔n though? @14:55?
@bjornfeuerbacher5514
@bjornfeuerbacher5514 Жыл бұрын
Yes, that part was false. The correction starts at about 16:45.
@GirishManjunathMusic
@GirishManjunathMusic Жыл бұрын
@@bjornfeuerbacher5514 yea I noticed.
@Jivvi
@Jivvi Жыл бұрын
Between 4 and 8, there are two: 5 and 7, but then between 5 and 10 there is only one: 7. When is last time there is only one prime between 𝑛 and 2𝑛, or are there infinitely many?
what fractions dream of
15:34
Michael Penn
Рет қаралды 13 М.
just an average recursion...OR IS IT?
18:24
Michael Penn
Рет қаралды 53 М.
This is not my neighbor  Terrible neighbor! #funny #zoonomaly #memes
00:26
Smart Sigma Kid #funny #sigma #comedy
00:26
CRAZY GREAPA
Рет қаралды 12 МЛН
The most useful polynomials you have never heard of.
20:26
Michael Penn
Рет қаралды 21 М.
Legendre's Conjecture is PROBABLY TRUE, and here's why
15:00
Michael Penn
Рет қаралды 27 М.
Transcendental Numbers - Numberphile
13:41
Numberphile
Рет қаралды 2 МЛН
a nice floor problem from the IMO long list.
18:47
Michael Penn
Рет қаралды 13 М.
Roger Penrose explains Godel's incompleteness theorem in 3 minutes
3:39
INCORRECT PROOF of Fermat's Last Theorem
10:20
Michael Penn
Рет қаралды 45 М.
Witness Numbers (and the truthful 1,662,803) - Numberphile
16:46
Numberphile
Рет қаралды 434 М.
A nice number theory problem from an Argentinian math olympiad
11:04
Russell's Paradox - a simple explanation of a profound problem
28:28
Jeffrey Kaplan
Рет қаралды 7 МЛН
You should know this abstract algebra exercise
19:59
Michael Penn
Рет қаралды 46 М.
This is not my neighbor  Terrible neighbor! #funny #zoonomaly #memes
00:26